matheraum.de
Raum für Mathematik
Offene Informations- und Nachhilfegemeinschaft

Für Schüler, Studenten, Lehrer, Mathematik-Interessierte.
Hallo Gast!einloggen | registrieren ]
Startseite · Forum · Wissen · Kurse · Mitglieder · Team · Impressum
Forenbaum
^ Forenbaum
Status Mathe
  Status Schulmathe
    Status Primarstufe
    Status Mathe Klassen 5-7
    Status Mathe Klassen 8-10
    Status Oberstufenmathe
    Status Mathe-Wettbewerbe
    Status Sonstiges
  Status Hochschulmathe
    Status Uni-Analysis
    Status Uni-Lin. Algebra
    Status Algebra+Zahlentheo.
    Status Diskrete Mathematik
    Status Fachdidaktik
    Status Finanz+Versicherung
    Status Logik+Mengenlehre
    Status Numerik
    Status Uni-Stochastik
    Status Topologie+Geometrie
    Status Uni-Sonstiges
  Status Mathe-Vorkurse
    Status Organisatorisches
    Status Schule
    Status Universität
  Status Mathe-Software
    Status Derive
    Status DynaGeo
    Status FunkyPlot
    Status GeoGebra
    Status LaTeX
    Status Maple
    Status MathCad
    Status Mathematica
    Status Matlab
    Status Maxima
    Status MuPad
    Status Taschenrechner

Gezeigt werden alle Foren bis zur Tiefe 2

Navigation
 Startseite...
 Neuerdings beta neu
 Forum...
 vorwissen...
 vorkurse...
 Werkzeuge...
 Nachhilfevermittlung beta...
 Online-Spiele beta
 Suchen
 Verein...
 Impressum
Das Projekt
Server und Internetanbindung werden durch Spenden finanziert.
Organisiert wird das Projekt von unserem Koordinatorenteam.
Hunderte Mitglieder helfen ehrenamtlich in unseren moderierten Foren.
Anbieter der Seite ist der gemeinnützige Verein "Vorhilfe.de e.V.".
Partnerseiten
Dt. Schulen im Ausland: Mathe-Seiten:Weitere Fächer:

Open Source FunktionenplotterFunkyPlot: Kostenloser und quelloffener Funktionenplotter für Linux und andere Betriebssysteme
StartseiteMatheForenKombinatorikWahrscheinlichkeitsraum
Foren für weitere Schulfächer findest Du auf www.vorhilfe.de z.B. Deutsch • Englisch • Französisch • Latein • Spanisch • Russisch • Griechisch
Forum "Kombinatorik" - Wahrscheinlichkeitsraum
Wahrscheinlichkeitsraum < Kombinatorik < Stochastik < Oberstufe < Schule < Mathe < Vorhilfe
Ansicht: [ geschachtelt ] | ^ Forum "Kombinatorik"  | ^^ Alle Foren  | ^ Forenbaum  | Materialien

Wahrscheinlichkeitsraum: Zeige, dass P(AuBuC) = ...
Status: (Frage) beantwortet Status 
Datum: 15:12 Fr 01.03.2019
Autor: bondi

Hallo,
wir sollen zeigen, dass

[mm] $P(A\cup B\cup [/mm] C) = [mm] P(A)+P(B)+P(C)-P(A\cap B)-P(A\cap C)-P(B\cap C)+P(A\cap B\cap [/mm] C)$ gilt.

Für zwei Wahrscheinlichkeiten habe ich das ganz gut mit dem Venn-Diagramm gelöst bekommen.

Ich habs so gemacht:

[mm] $P(A\cup [/mm] B) = [mm] P(A)+P(B)-P(A\cap [/mm] B)$

$P(A)$ kann ich auch so darstellen: [mm] $P(A\setminus [/mm] B)+P(B)$

Das eingesetzt ergibt:

[mm] $P(A\cup [/mm] B) =  [mm] P(A\setminus [/mm] B)+P(B)$
[mm] $P(A\cup [/mm] B) = [mm] P(A)-P(A\cap [/mm] B)+P(B)$

Sortieren:

[mm] $P(A\cup [/mm] B) = [mm] P(A)+P(B)-P(A\cap [/mm] B)$

Jetzt aber sollen wir zeigen, dass

[mm] $P(A\cup B\cup C)=P(A)+P(B)+P(C)-P(A\cap B)-P(A\cap C)-P(B\cap C)+P(A\cap B\cap [/mm] C)$

ist. Mit drei Wahrscheinlichkeiten komme ich noch nicht klar. Reicht es eigentlich, wenn ich ein Element auf andere Weise darstelle?

Ich habe diese Frage in keinem Forum auf anderen Internetseiten gestellt.





        
Bezug
Wahrscheinlichkeitsraum: Mitteilung
Status: (Mitteilung) Reaktion unnötig Status 
Datum: 15:18 Fr 01.03.2019
Autor: bondi

Ich habe ein paar Fehler in meiner Frage entdeckt. Kann ich das was ich abgeschickt habe nachträglich ändern?

Bezug
                
Bezug
Wahrscheinlichkeitsraum: bearbeiten
Status: (Mitteilung) Reaktion unnötig Status 
Datum: 15:20 Fr 01.03.2019
Autor: Loddar

Hallo bondi,

[willkommenmr] !!

Unterhalb des Artikels sollte einer der Buttons lauten:

[Dateianhang nicht öffentlich]

Mit [4] kannst Du Deine Fragen editieren.


Gruß
Loddar

Dateianhänge:
Anhang Nr. 1 (Typ: PNG) [nicht öffentlich]
Bezug
        
Bezug
Wahrscheinlichkeitsraum: Antwort
Status: (Antwort) fertig Status 
Datum: 17:51 Fr 01.03.2019
Autor: Fulla

Hallo bondi,

ich habe die Formeln in deinem Frage-Artikel so bearbeitet, dass sie richtig dargestellt werden. Klicke auf eine Formel oder sieh dir den Quelltext an, wenn du wissen willst, wie die entsprechenden Befehle lauten.

> Hallo,
> wir sollen zeigen, dass

>

> [mm]P(A\cup B\cup C) = P(A)+P(B)+P(C)-P(A\cap B)-P(A\cap C)-P(B\cap C)+P(A\cap B\cap C)[/mm]
> gilt.

>

> Für zwei Wahrscheinlichkeiten habe ich das ganz gut mit
> dem Venn-Diagramm gelöst bekommen.

>

> Ich habs so gemacht:

>

> [mm]P(A\cup B) = P(A)+P(B)-P(A\cap B)[/mm]

>

> [mm]P(A)[/mm] kann ich auch so darstellen: [mm]P(A\setminus B)+P(B)[/mm]

Hier hast du einen (Tipp-)Fehler gemacht. Links muss [mm]P(A\cup B)[/mm] stehen. Unten hast du es richtig geschrieben.

> Das eingesetzt ergibt:

>

> [mm]P(A\cup B) = P(A\setminus B)+P(B)[/mm]
> [mm]P(A\cup B) = P(A)-P(A\cap B)+P(B)[/mm]

>

> Sortieren:

>

> [mm]P(A\cup B) = P(A)+P(B)-P(A\cap B)[/mm]

>

> Jetzt aber sollen wir zeigen, dass

>

> [mm]P(A\cup B\cup C)=P(A)+P(B)+P(C)-P(A\cap B)-P(A\cap C)-P(B\cap C)+P(A\cap B\cap C)[/mm]

>

> ist. Mit drei Wahrscheinlichkeiten komme ich noch nicht
> klar. Reicht es eigentlich, wenn ich ein Element auf andere
> Weise darstelle?

Wie so oft, hilft hier zunächst eine Skizze, z.B. []diese. Da geht es zwar um Mengen und nicht um Wahrscheinlichkeiten, aber prinzipiell geht es um dasselbe.

[mm]P(A\cup B\cup C)[/mm] wird hier durch die Gesamtfläche der drei Kreise beschrieben.
Mit dem Ansatz [mm]P(A)+P(B)+P(C)[/mm] hast du aber zuviel genommen, da sich die Kreise im Allgemeinen überlappen. Z.B. hast du den linsenförmigen Bereich [mm]P(A\cap B)[/mm] einmal bei [mm]P(A)[/mm] und einmal bei [mm]P(B)[/mm] mitgezählt, den musst du also am Ende einmal abziehen.

Führe diesen Gedankengang mit anderen Bereichen weiter und überlege dir, wie oft dabei der "dreieckige" Bereich [mm]P(A\cap B\cap C)[/mm] dazugezählt und abgezogen wird und korrigiere das entsprechend, so dass jeder Bereich genau einmal addiert wird.


Die Variante mit Mengendifferenzen, die du oben ansprichst, habe ich nicht durchgespielt. Das funktioniert bestimmt auch, wird aber ungleich komplizierter als der von mir beschriebene Weg.

Stör dich nicht daran, dass in dem Beweis nicht jeder Schritt mit einer Formel "belegt" wird. Es ist völlig legitim, Beweisargumente "mit Worten" zu beschreiben (solange die Formulierung nicht schwammig ist).


Lieben Gruß,
Fulla

Bezug
        
Bezug
Wahrscheinlichkeitsraum: Antwort
Status: (Antwort) fertig Status 
Datum: 19:30 Sa 02.03.2019
Autor: Gonozal_IX

Hiho,

> Für zwei Wahrscheinlichkeiten habe ich das ganz gut […] gelöst bekommen.

mehr brauchst du doch gar nicht!
Es ist doch $P(A [mm] \cup [/mm] B [mm] \cup [/mm] C) = P(A [mm] \cup [/mm] D)$ mit $D = B [mm] \cup [/mm] C$.

Nun hast du nur noch zwei Mengen und wendest dein Wissen an.
Dann ersetzt du D wieder durch $B [mm] \cup [/mm] C$ und machst weiter...

Gruß,
Gono


Bezug
        
Bezug
Wahrscheinlichkeitsraum: Frage (beantwortet)
Status: (Frage) beantwortet Status 
Datum: 08:33 Di 05.03.2019
Autor: bondi

Hi.

Nur um sicher zu sein, dass ich das richtig verstanden habe:

[mm] $P(A\cup B\cup [/mm] C)$

ist das Gleiche wie

[mm] $P(A\cup B)\cup [/mm] P(B [mm] \cup [/mm] C)$

Nachweis Teil 1:

[mm] $P(A\cup [/mm] B) = P(A) + P(B) - [mm] P(A\cap [/mm] B)$


$P(A) = [mm] P(A\setminus [/mm] B) + [mm] P(A\cap [/mm] B)$

[mm] $P(A\setminus [/mm] B) = P(A) - [mm] P(A\cup [/mm] B)$


[mm] $P(B\setminus [/mm] A) = P(B) - [mm] P(A\cup [/mm] B)$

$P(B) = [mm] P(B\setminus [/mm] A) + [mm] P(A\cap [/mm] B)$


Daraus folgt:

[mm] $P(A\cup [/mm] B) = [mm] P(A\setminus [/mm] B) + [mm] P(A\cap [/mm] B) + [mm] P(B\setminus [/mm] A) + [mm] P(A\cap [/mm] B) - [mm] P(A\cap [/mm] B) = [mm] P(A\setminus [/mm] B) + [mm] P(A\cap [/mm] B) + [mm] P(B\setminus [/mm] A)$


Nachweis Teil 2:

[mm] $P(B\cup [/mm] C) = P(B) + P(C) - [mm] P(B\cap [/mm] C)$


$P(B) = [mm] P(B\setminus [/mm] C) + [mm] P(B\cap [/mm] B)$

[mm] $P(B\setminus [/mm] C) = P(B) - [mm] P(B\cup [/mm] C)$


$P(C) = [mm] P(C\setminus [/mm] B) + [mm] P(B\cap [/mm] C)$

[mm] $P(C\setminus [/mm] B) = P(C) - [mm] P(B\cup [/mm] C)$


Daraus folgt:

[mm] $P(B\cup [/mm] C) = [mm] P(B\setminus [/mm] C) + [mm] P(B\cap [/mm] C) + [mm] P(C\setminus [/mm] B) + [mm] P(B\cap [/mm] C) - [mm] P(B\cap [/mm] C) =  [mm] P(B\setminus [/mm] C) + [mm] P(B\cap [/mm] C) + [mm] P(C\setminus [/mm] B)$


Somit ist

[mm] $P(A\cup [/mm] B) [mm] \cup P(B\cup [/mm] C) = [mm] P(A\cup B\cup [/mm] C)$

Für ein Feedback bin ich dankbar.

Bezug
                
Bezug
Wahrscheinlichkeitsraum: Mitteilung
Status: (Mitteilung) Reaktion unnötig Status 
Datum: 09:06 Di 05.03.2019
Autor: fred97


> Hi.
>  
> Nur um sicher zu sein, dass ich das richtig verstanden
> habe:

Ich glaube, dass Du in der falschen Diskussion gelandet bist. Das was Du unten schreibst hat mit der Frage nach den Ehepaaren nix zu tun.

>  
> [mm]P(A\cup B\cup C)[/mm]
>  
> ist das Gleiche wie
>  
> [mm]P(A\cup B)\cup P(B \cup C)[/mm]
>
> Nachweis Teil 1:
>  
> [mm]P(A\cup B) = P(A) + P(B) - P(A\cap B)[/mm]
>  
>
> [mm]P(A) = P(A\setminus B) + P(A\cap B)[/mm]
>  
> [mm]P(A\setminus B) = P(A) - P(A\cup B)[/mm]
>  
>
> [mm]P(B\setminus A) = P(B) - P(A\cup B)[/mm]
>  
> [mm]P(B) = P(B\setminus A) + P(A\cap B)[/mm]
>  
>
> Daraus folgt:
>  
> [mm]P(A\cup B) = P(A\setminus B) + P(A\cap B) + P(B\setminus A) + P(A\cap B) - P(A\cap B) = P(A\setminus B) + P(A\cap B) + P(B\setminus A)[/mm]
>  
>
> Nachweis Teil 2:
>  
> [mm]P(B\cup C) = P(B) + P(C) - P(B\cap C)[/mm]
>  
>
> [mm]P(B) = P(B\setminus C) + P(B\cap B)[/mm]
>  
> [mm]P(B\setminus C) = P(B) - P(B\cup C)[/mm]
>  
>
> [mm]P(C) = P(C\setminus B) + P(B\cap C)[/mm]
>  
> [mm]P(C\setminus B) = P(C) - P(B\cup C)[/mm]
>  
>
> Daraus folgt:
>  
> [mm]P(B\cup C) = P(B\setminus C) + P(B\cap C) + P(C\setminus B) + P(B\cap C) - P(B\cap C) = P(B\setminus C) + P(B\cap C) + P(C\setminus B)[/mm]
>  
>
> Somit ist
>
> [mm]P(A\cup B) \cup P(B\cup C) = P(A\cup B\cup C)[/mm]
>  
> Für ein Feedback bin ich dankbar.


Bezug
                        
Bezug
Wahrscheinlichkeitsraum: verschoben
Status: (Mitteilung) Reaktion unnötig Status 
Datum: 13:54 Di 05.03.2019
Autor: Loddar

Hallo!


Ich habe die Frage mal in den vermeintlich korrekten Thread verschoben.


Gruß
Loddar

Bezug
                
Bezug
Wahrscheinlichkeitsraum: Antwort
Status: (Antwort) fertig Status 
Datum: 07:30 Do 07.03.2019
Autor: Gonozal_IX

Hiho,

> Hi.
>  
> Nur um sicher zu sein, dass ich das richtig verstanden
> habe:
>  
> [mm]P(A\cup B\cup C)[/mm]
>  
> ist das Gleiche wie
>  
> [mm]P(A\cup B)\cup P(B \cup C)[/mm]

Der zweite Ausdruck ist doch Unfug, wie willst du zwei reelle Zahlen vereinigen?

Du wolltest $P(A [mm] \cup [/mm] B [mm] \cup [/mm] C)$ berechnen.
Nun war mein Hinweis: Setze $D := B [mm] \cup [/mm] C$, dann steht da:
$P(A [mm] \cup [/mm] B [mm] \cup [/mm] C) = P(A [mm] \cup [/mm] D)$

Da hat sich gar nichts geändert, einzig dass du jetzt vermeindlich wieder nur zwei Mengen vereinigst und damit deine bekannte Regel anwenden kannst, also:

[mm] $=\blue{P(A)} [/mm] + [mm] \red{P(D)} [/mm] - [mm] \green{P(A\cap D)}$ [/mm]

Setzen wir nun D wieder ein, erhalten wir:

[mm] $=\blue{P(A)} [/mm] + [mm] \red{P(B \cup C)} [/mm] - [mm] \green{P(A \cap (B \cup C))}$ [/mm]

Anwenden der Morganschen Regel und deiner Rechenregel für zwei Mengen ergibt:

[mm] $=\blue{P(A)} [/mm] + [mm] \red{P(B) + P(C) - P(B\cap C)} [/mm] - [mm] \green{P((A \cap B) \cup (A \cap C))}$ [/mm]

Das vorne sieht ja schon gut aus und im letzten Ausdruck steht eigentlich auch nur wieder die Vereinigung von zwei Mengen, das sehen wir besser, wenn wir $E = (A [mm] \cap [/mm] B)$ und $F=(A [mm] \cap [/mm] C)$ setzen, das ergibt nämlich:

$=P(A) + P(B) + P(C) - P(B [mm] \cap [/mm] C) - [mm] \green{P(E \cup F)}$ [/mm]

und wieder wenden wir die bekannte Regel für zwei Mengen an und erhalten:

$=P(A) + P(B) + P(C) - P(B [mm] \cap [/mm] C) - [mm] \green{\left(P(E) + P(F) - P(E \cap F)\right)}$ [/mm]

Setzen wir wieder die Ausdrücke von E und F ein, erhalten wir:

$=P(A) + P(B) + P(C) - P(B [mm] \cap [/mm] C) - [mm] \green{\left(P(A \cap B) + P(A \cap C) - P(A \cap B \cap A \cap C)\right)}$ [/mm]

Klammer auflösen, umsortieren und beachten, dass zweimal mit der selben Menge schneiden egal ist, gibt:

$=P(A) + P(B) + P(C) - [mm] \green{P(A\cap B) - P(A\cap C)} [/mm] - P(B [mm] \cap [/mm] C) + [mm] \green{P(A \cap B \cap C)}$ [/mm]

Gruß,
Gono





Bezug
                        
Bezug
Wahrscheinlichkeitsraum: Mitteilung
Status: (Mitteilung) Reaktion unnötig Status 
Datum: 10:32 Do 07.03.2019
Autor: bondi

Hi Gono,
vielen Dank für deine farbenfrohe Erklärung. Hat mir sehr geholfen.

Viele Grüße,
bondi

Bezug
Ansicht: [ geschachtelt ] | ^ Forum "Kombinatorik"  | ^^ Alle Foren  | ^ Forenbaum  | Materialien


^ Seitenanfang ^
www.matheraum.de
[ Startseite | Forum | Wissen | Kurse | Mitglieder | Team | Impressum ]